- PowerScore Staff
- Posts: 5972
- Joined: Mar 25, 2011
- Wed Nov 08, 2017 5:33 pm
#41387
Complete Question Explanation
The correct answer choice is (B).
The third rule stipulates that M must be presented earlier than both O and P. The question stem then establishes that P is presented at 2 PM. Thus, combining this information and the third rule, we can conclude that M must be presented earlier than 2 PM, which only leaves 1 PM. This is what answer choice (B) states, and thus (B) is correct.
Answer choice (A): G could be presented at 3 PM, but as this hypothetical shows, does not have to be:
For example, we could sort the variables as follows:
West: M, P, S
East: U, G, O
Answer choice (B): This is the correct answer choice.
Answer choice (C): O could be presented at 2 PM, but as this hypothetical shows, does not have to be:
For example, we could sort the variables as follows:
West: M, P, S
East: U, G, O
Answer choice (D): While S could be presented at 1 PM, it does not have to be. The group of available variables for 1 PM consists of M, S, and U. M can be presented in either the East or West theater, and depending on where M is played, the other theater will present S or U. Thus, either S or U could be presented at 1, but neither must be presented at 1.
Answer choice (E): While S could be presented at 1 PM, it does not have to be. The group of available variables for 1 PM consists of M, S, and U. M can be presented in either the East or West theater, and depending on where M is played, the other theater will present S or U. Thus, either S or U could be presented at 1, but neither must be presented at 1.
This hypothetical shows that U does not have be presented at 1PM:
West: S P G
East: M U O
The correct answer choice is (B).
The third rule stipulates that M must be presented earlier than both O and P. The question stem then establishes that P is presented at 2 PM. Thus, combining this information and the third rule, we can conclude that M must be presented earlier than 2 PM, which only leaves 1 PM. This is what answer choice (B) states, and thus (B) is correct.
Answer choice (A): G could be presented at 3 PM, but as this hypothetical shows, does not have to be:
For example, we could sort the variables as follows:
West: M, P, S
East: U, G, O
Answer choice (B): This is the correct answer choice.
Answer choice (C): O could be presented at 2 PM, but as this hypothetical shows, does not have to be:
For example, we could sort the variables as follows:
West: M, P, S
East: U, G, O
Answer choice (D): While S could be presented at 1 PM, it does not have to be. The group of available variables for 1 PM consists of M, S, and U. M can be presented in either the East or West theater, and depending on where M is played, the other theater will present S or U. Thus, either S or U could be presented at 1, but neither must be presented at 1.
Answer choice (E): While S could be presented at 1 PM, it does not have to be. The group of available variables for 1 PM consists of M, S, and U. M can be presented in either the East or West theater, and depending on where M is played, the other theater will present S or U. Thus, either S or U could be presented at 1, but neither must be presented at 1.
This hypothetical shows that U does not have be presented at 1PM:
West: S P G
East: M U O
Dave Killoran
PowerScore Test Preparation
Follow me on X/Twitter at http://twitter.com/DaveKilloran
My LSAT Articles: http://blog.powerscore.com/lsat/author/dave-killoran
PowerScore Podcast: http://www.powerscore.com/lsat/podcast/
PowerScore Test Preparation
Follow me on X/Twitter at http://twitter.com/DaveKilloran
My LSAT Articles: http://blog.powerscore.com/lsat/author/dave-killoran
PowerScore Podcast: http://www.powerscore.com/lsat/podcast/